LSAT and Law School Admissions Forum

Get expert LSAT preparation and law school admissions advice from PowerScore Test Preparation.

 Administrator
PowerScore Staff
  • PowerScore Staff
  • Posts: 8950
  • Joined: Feb 02, 2011
|
#22936
Complete Question Explanation

Assumption. The correct answer choice is (E)

This stimulus concludes that some cases of high blood pressure are not stress-induced, because illnesses caused by stress can only be treated by reducing it, and some cases of high BP can be effectively treated with medicine. The argument can be summarized in the following way:
  • Premise (1): ..... SII (stress-induced illness) ..... :arrow: ..... TRS (treatment must reduce stress)

    Premise (2): ..... HBP ..... :some: ..... M (some high blood pressure is treatable with medication)

    Conclusion: ..... HBP ..... :some: ..... SII (some high blood pressure is not stress-induced)

Notice the inconsistency between the premises and the conclusion: just because some instances of high blood pressure are treatable with medicine, it does not necessarily follow that they are not stress-induced. What if the medicine actually worked by reducing stress? The conclusion only follows if we assume that the medication used to treat some cases of high BP does not work by reducing stress (M :arrow: TRS). This is precisely what the author states in answer choice (E).

Answer choice (A): While this answer choice supports the conclusion by advancing the notion that there is no causal link between stress and high BP, it is not an assumption required by the argument. It is only necessary that some cases of high BP are unrelated to stress, not all of them. Beware of answer choices that are too strong in the context of Assumption questions.

Answer choice (B): This answer choice weakens the conclusion and is incorrect.

Answer choice (C): Even if reduced stress reduced a person's responsiveness to medicine, the conclusion of the argument will not change. Since the argument does not require it, answer choice (C) is incorrect.

Answer choice (D): This may seem to be an attractive answer at first. Let's try the Assumption Negation technique: what if all conditions that are treatable by medicines are also treatable through the reduction of stress? Wouldn't this be consistent with the idea that all cases of high BP are potentially caused by stress? Not necessarily.

Remember that the necessary condition in the author's first premise requires that any illness caused by stress is treatable only by the reduction of stress. Having two effective treatments at our disposal (medicine and stress reduction) still means that the illness in question is not stress-induced. Because the conclusion is not weakened by the logical opposite of answer choice (D), this answer choice is not a necessary assumption for it.

Answer choice (E): This is the correct answer choice. See discussion above.
 Lina
  • Posts: 22
  • Joined: Aug 05, 2013
|
#10184
Hello, I took the Powerscore online course and am reviewing from my course book. I was hoping you could help me work out some problems from Chapter 5, Assumption

Q14: I chose A and afterward realized that they were not dealing with correlations, is this why it is wrong ? Also, I negated it to mean that if stress and HBP are not coincidental, it shows that stress causes HPB and can be cured by stress reduction.

Thanks for your help!
 David Boyle
PowerScore Staff
  • PowerScore Staff
  • Posts: 836
  • Joined: Jun 07, 2013
|
#10201
Lina wrote:**Please note, this question has been split into 3 so we can better assist you!**

Hello, I took the Powerscore online course and am reviewing from my course book. I was hoping you could help me work out some problems from Chapter 5, Assumption

Pg 5-42 Q14: I chose A and afterward realized that they were not dealing with correlations, is this why it is wrong ? Also, I negated it to mean that if stress and HBP are not coincidental, it shows that stress causes HPB and can be cured by stress reduction.

Thanks for your help!
Dear Lina:

The passage may make some hypothetical correlation between--or at least raises questions about--HBP and stress, i.e., that it is thinkable that stress could cause some high blood pressure. (Though it says it in a negative way, i.e., that some HBP may not be caused by stress.) --But even if you negated answer choice A, then it would say that there may be some non-coincidental correlation between stress and HBP. This dos not per se mean that stress *causes* HBP, though, since causation and correlation are different.
Hope that helps,

David
 Blueballoon5%
  • Posts: 156
  • Joined: Jul 13, 2015
|
#21680
I got the right answer, but I am confused with the answer explanation for answer D. The answer key explains that, "Let's try the Assumption Negation technique: what if all conditions that are treatable by medicines are also treatable through the reduction of stress?" However, this does not seem like the logical opposite of the stimulus. The stimulus contains the word "some," and the logical opposite of this term is "none"... right?

Hope you can help me :)
 Laura Carrier
PowerScore Staff
  • PowerScore Staff
  • Posts: 38
  • Joined: Oct 04, 2015
|
#21683
Hi Blueballoon,

You are absolutely right that the logical opposite of “some” is “none.” And thus you are also correct to point out that a literal application of the Assumption Negation Technique to answer choice (D) would be to replace “some” with “none” so that it would read more or less as follows:
None of the conditions that are treated effectively by medicines are not also treatable through the reduction of stress.
But since that direct negation leaves us with a double negative, which is always a bit challenging to interpret, the online explanation for this answer choice goes a step further and simplifies the resulting negation to remove that double negative, in order to make it easier to see its impact on the stimulus.

What answer choice (D) is really telling us by using “some” is that there is at least one condition treatable by medicine that is not treatable by stress reduction. Although it is true that the most straightforward way of logically negating this is to say:
there is not even one condition (i.e., “none”) treatable by medicine that is not treatable by stress reduction
this is also logically equivalent to saying what the online explanation offers, which is that:
all conditions treatable by medicine are treatable by stress reduction
If there isn’t even one MTC (medically treatable condition) that is not treatable by stress reduction, then it is logically equivalent to say that all MTCs are treatable by stress reduction. This version just makes it easier to check answer choice (D) against the stimulus to see what impact its negation has on the conclusion.

I hope that clarifies things!
Laura
 yrivers
  • Posts: 68
  • Joined: Mar 15, 2017
|
#35874
Dear PowerScore,

Following up on the last post re: answer D, I wanted one clarification. If the Assumption Negation for D tells us that "all conditions treatable by medicine are treatable by stress reduction," doesn't that make the argument fall apart? Because then it contradicts the last part of the stimulus that notes, "...some cases of high blood pressure must not be caused by stress"?

Bit confused; would appreciate your clarification! Thank you!

Yaesul
 Adam Tyson
PowerScore Staff
  • PowerScore Staff
  • Posts: 5374
  • Joined: Apr 14, 2011
|
#36063
That negation makes answer D look particularly attractive, Yaesul, but it actually requires that you make a classic mistake of conditional reasoning, a Mistaken Reversal.

Your analysis of D is good - the negation says that if you can treat something with medicine (such as high blood pressure) then you can also treat it with stress reduction. But does that mean that stress caused the illness? Not necessarily! Let's diagram the conditional claim in the argument:

Treatable Illness Caused By Stress :arrow: Treatable Only by Stress Reduction

Answer D, negated as you have done, is:

Treatable by Stress Reduction :arrow: Caused By Stress

Whoops! Can't do that, can we?

We can also analyze it purely causally - the reduction of the cause correlating well with the reduction of the effect strengthens, but does not by itself prove, a causal claim. If you want to suggest that stress may in fact be the cause of high blood pressure, then showing that stress reduction goes hand in hand with less high blood pressure is a good start but not definitive proof. The opposite argument, that stress is not always the cause, is weakened but not destroyed.

And finally, looked at another way, just because you can treat it be reducing stress doesn't mean stress caused it. That's like saying treating it with medicine proves that it was caused by not taking medicine!

I like the way you were thinking there, Yaesul! Give this some further thought, though, and keep up the good work.
 lunsandy
  • Posts: 61
  • Joined: Oct 14, 2017
|
#41015
Hi Adam + Powerscore,

I am still confused with D. You said D is wrong because of Mistaken Reversal. When I diagramed it I got:

Stim:
Treatable Illness Caused By Stress :arrow: Treatable Only by Stress Reduction

Answer D:
No conditions that are treated effectively by medicines are not also treatable thorugh the reduction of stress :arrow: All conditions that are treated effectively by medicines are also treatable through the reduction of stress.

Treatable effectively by medicine :arrow: Treatable through the reduction of Stress. Isn't that the same as answer E?

Answer E:
Medicine used to treat HBP :arrow: Medicine reduce stress
 Francis O'Rourke
PowerScore Staff
  • PowerScore Staff
  • Posts: 471
  • Joined: Mar 10, 2017
|
#41126
Hi Lunsandy,

The logical negation of answer choice (E) would tell us that a medicine that treats high blood pressure reduces stress. In other words, the medicines themselves cause a reduction in stress which might be what treats the illness.

The negation of answer choice (D) would tell us that if you can effectively treat an illness with medicine (such as high blood pressure), then you can also treat it by reducing stress. This is not saying that these medicines work by reducing stress, but rather that we have other options in treating the illness.

Let me know if this helps.
 Blueballoon5%
  • Posts: 156
  • Joined: Jul 13, 2015
|
#44674
Francis O'Rourke wrote: The logical negation of answer choice (E) would tell us that a medicine that treats high blood pressure reduces stress. In other words, the medicines themselves cause a reduction in stress which might be what treats the illness.

The negation of answer choice (D) would tell us that if you can effectively treat an illness with medicine (such as high blood pressure), then you can also treat it by reducing stress. This is not saying that these medicines work by reducing stress, but rather that we have other options in treating the illness.
Hi Francis. I hope you (or someone else) can clarify some things.

I broke down the stimulus for this question into parts:
Premise 1: "Some HBP treated by medicine"
Premise 2: "Caused by stress :arrow: Treated by stress-reduction"
Conclusion: "Some HBP not caused by stress"

When examining the stimulus above, I realize Premise 1 and the Conclusion is the contrapositive of Premise 2, which is "NOT treated by stress-reduction :arrow: NOT caused by stress." This is very similar to "Some HBP treated by medicine" (premise 1) :arrow: "Some HBP not caused by stress" (conclusion).

Therefore, this seems like a solid argument, right? And thus, answer choice E is correct because it simply eliminates one possible alternative, that the medicine actually reduced both the HBP and stress.

For confirmation, using the Assumption Negation technique, the negated version of answer choice E effectively attacks the stimulus because it attacks the necessary condition of premise 2. If medicine does treat stress, then it makes the necessary condition in premise 2 not actually required. Since it attacks this conditional statement (which the conclusion relies on), the argument falls apart.

I am not sure if my thinking is correct in the bold above. However, if I am correct, I am a little confused why answer choice D wouldn't also apply here. If we negate answer choice D, we also have a situation where the "treatment of stress-reduction" is actually not necessary for the stress-caused condition. If all conditions, including HBP, can be treated by two ways, doesn't that undermine Premise 2?

Hope you can help!

Get the most out of your LSAT Prep Plus subscription.

Analyze and track your performance with our Testing and Analytics Package.